LSAT and Law School Admissions Forum

Get expert LSAT preparation and law school admissions advice from PowerScore Test Preparation.

 Adam Tyson
PowerScore Staff
  • PowerScore Staff
  • Posts: 5400
  • Joined: Apr 14, 2011
|
#4599
Let me see if I can help here.

The author's argument starts with the premise that when lava solidifies, it magnetizes in the same direction as the Earth's magnetic field. What he never says (and therefore assumes) is that it never changes after that - that once it's solid, it continues to be magnetically orientated the way it was when it started.

If he included a new premise that said "no solidified lava ever changes the direction of its magnetic field", that would make his conclusion much better, in my opinion, and apparently in yours. But just because that would be a great premise, is it necessary that he assume that? Could his argument still be valid if sometimes lava did change it's magnetic direction? Sure - it just just depends on how much. 1% of the time and the argument is still pretty good. 99% of the time, and it stinks.

The stem asks us which assumption is required, and C is absolutely the best of the bunch. You want to know why they didn't include a stronger assumption - that it never changes - to make the argument better? The answer is that an assumption that strong is not required for his argument to work. Sure, it would help a lot if it were true, but it isn't essential. That's why it wrecks the argument, and doesn't just hurt it, when we negate it.

As an experiment, try negating the more extreme assumption you are looking for - no lava changes magnetic direction becomes some lava does. Is the argument weakened? Perhaps. Destroyed? Nope - "some" isn't powerful enough to do that.

Did that help?

Adam M. Tyson
PowerScore LSAT Instructor
 moshei24
  • Posts: 465
  • Joined: Mar 20, 2012
|
#4614
So even if the correct answer assumption CAN hurt the argument, but doesn't for sure, it's the assumption that works? This is the first time that I've seen a question like this. I'm guessing questions like this one don't occur too often?
 Adam Tyson
PowerScore Staff
  • PowerScore Staff
  • Posts: 5400
  • Joined: Apr 14, 2011
|
#4616
Try looking at it the other way around, Moshei - if the negated form of the answer weakens the argument without destroying it, it's wrong. You want the negated assumption to completely destroy the argument. The assumption itself should help the argument, but it doesn't have to help very much. Try this example, which I use in my class all the time:

Argument: My friend Hugh is a great tennis player

Assumption: Sometimes he manages to hit the ball over the net

Does that assumption help the argument? A little. Does it leave room for the possibility that sometimes he doesn't hit the ball over the net? Yup. Does that possibility have the potential to weaken the argument? It sure does. But consider the negation:

Hugh NEVER manages to hit the ball over the net

What effect does that have on my conclusion that he's great? It destroys it.

Did I have to assume that he ALWAYS hits the ball over the net? No, because while that statement, if true, would be very helpful, it's still possible that he might sometimes fail to get the ball over the net and yet still be great. The negation there would be:

Hugh doesn't always hit the ball over the net

That weakens, but doesn't come close to destroying, my argument.

Assumptions don't have to justify the conclusion. They don't have to help much at all. They only have to be statements that, if the conclusion is true, must also be true. They can be very, very weak.
 moshei24
  • Posts: 465
  • Joined: Mar 20, 2012
|
#4618
I see.

So for a justify question that wouldn't work?

Thanks.

-Moshe
 Adam Tyson
PowerScore Staff
  • PowerScore Staff
  • Posts: 5400
  • Joined: Apr 14, 2011
|
#4619
That's right - a justify answer has a different relationship with the stimulus than an assumption. The justify has to prove the conclusion is true, to make it unassailable. We don't use any version of the negation technique on justify answers.

You can look at the two types of questions as calling for opposing conditional relationships, if that helps. An assumption is:

Conclusion True -> Assumption True (and that's why the negation technique works - it's the contrapositive of that relationship)

A justify answer is:

Answer True -> Conclusion True

For justify answers, you often want an extreme answer - something always happens, or never happens, or it must be the case that XYZ, etc. Whatever it takes to add to your premise(s) to make the conclusion a certainty, rather than a possibility or a probability.
 moshei24
  • Posts: 465
  • Joined: Mar 20, 2012
|
#4621
Gotcha. Thank you.
 Sdaoud17
  • Posts: 85
  • Joined: Apr 13, 2013
|
#9452
Can you please show to Negate Answer E ? Thank you
 Ron Gore
PowerScore Staff
  • PowerScore Staff
  • Posts: 220
  • Joined: May 15, 2013
|
#9464
Hi Sdaoud17,

To negate (E), you could say: "as lava flows down the side of a volcano, it might not pick up magnetized rocks"

Or, you could say: "as lava flows down the side of a volcano, it does not necessarily pick up magnetized rocks"

Hope this helps.

Ron
 vas
  • Posts: 10
  • Joined: Aug 05, 2015
|
#19435
Hello,

Would we be able to diagram the argument in order to identify the necessary assumption? I'm having problems understanding the actual cause and effect (and it assumes) as described in the answer explanations and think a diagram might help.

Thanks!
 Clay Cooper
PowerScore Staff
  • PowerScore Staff
  • Posts: 241
  • Joined: Jul 03, 2015
|
#19442
Hi Vas,

Thanks for your question.

The argument in the stimulus does not seem to me to lend itself to diagramming, because it does not employ conditional reasoning.

The conclusion that change in direction in earth's magnetic field takes place slowly over hundreds of thousands of years relies on the evidence that lava samples which solidified thousands of years apart nonetheless have very similar directions of polarity.

Answer choice C is correct because it introduces the (probably unforeseen) possibility that these lava formations had changed polarity for other reasons, after they had solidified. If this were true, it offers a plausible alternative cause for the observed effect - similar polarities in samples dried thousands of years apart - and thus the negated form of answer choice C becomes a powerful attack on the authors argument, and C is therefore the correct answer.

Does that clarify the argument and assumption in this question?

Get the most out of your LSAT Prep Plus subscription.

Analyze and track your performance with our Testing and Analytics Package.